Is f(a) < f(c) < f(b) true for a continuous and one-to-one function?

  • Thread starter xsw001
  • Start date
It makes much more sense now. In summary, we can prove that if a<c<b, then f(a)<f(c)<f(b) by contradiction, assuming f 1-1 and using the IVT to show that f(c) cannot be equal to f(a) or f(b), leading to a contradiction. Therefore, it must be that f(a)<f(c)<f(b).
  • #1
xsw001
37
0
Let f:[a,b]->R be continuous and one-to-one such that f(a)<f(b).
Let a<c<b. Prove that f(a)<f(c)<f(b)

My first instinct is to apply intermediate value theorem. Let me know whether my proof makes sense or not.

Proof:
Since f:[a,b]->R is continuous and one-to-one.
Therefore f is strictly increasing function.
Suppose a<c<b
According to Intermediate Value Theorem
There exists f(c) such that f(a)<f(c)<f(b)
 
Last edited:
Physics news on Phys.org
  • #2
The IVT only uses continuity and merely guarantees that there exists at least one c, such that a < c < b and f(a) < f(c) < f(b). This is not the same statement as what is to be proved, namely that f(a) < f(c) < f(b) for all c in [a,b]. So your proof is incomplete.

You will need to use that f is 1-1 and the fact that you say f is therefore strictly increasing. However it might be illuminating to study the proof of the IVT in order to adapt it to this situation.
 
  • #3
Okay, I'm still not completely getting the right approach to complete this proof. "fzero", you were saying that I need to use the fact of f is 1-1, along with f is continuous on [a, b] that only implies the function is NOT a constant function, so it is strictly monotone, isn't it?

The IVT states if f:[a,b]->R is continuous, and if f(a)<c<f(b) or f(b)<c<f(a), then there exists an x in (a,b) such that f(x)=c.

I'm a little unclear and confused as of how to approach it the right way, can you further clarify a little? Thanks.
 
Last edited:
  • #4
xsw001 said:
Okay, I'm still not completely getting the right approach to complete this proof. "fzero", you were saying that I need to use the face of f is 1-1, along with f is continuous on [a, b] that only implies the function is NOT a constant function, so it is strictly monotone, isn't it?

The IVT states if f:[a,b]->R is continuous, and if f(a)<c<f(b) or f(b)<c<f(a), then there exists an x in (a,b) such that f(x)=c.

I'm a little unclear and confused as of how to approach it the right way, can you further clarify a little? Thanks.

It's clear that f is strictly increasing, but I'm not sure that we've proven that with the sufficient rigor that might be expected in your course.

The problem with direct application of the IVT is that it starts with a u such that f(a) < u < f(b) and tells you that there's an x in [a,b] such that f(x) = u. In your conjecture we start with c such that a < c < b. The IVT doesn't directly say anything about f(c).

You can use the IVT to prove this by contradiction. Assume that f(c) > f(b) and apply IVT to the intervals [a,c] and [c,b]. This will lead to a contradiction with the fact that f is 1-1.
 
  • #5
Oh, ic, thanks!
 
  • #6
xsw001 said:
Oh, ic, thanks!

One more note on this. I forgot that you have to exclude f(c) < f(a), but I think that's straightforward too.
 
  • #7
Here is the sketch of the proof.
Assume by contradiction that a<c<b, and case 1) f(a)<f(b)<f(c) or case 2) f(c)<f(a)<f(b)
Since the function is one-to-one, therefore the graph of the continuous function can't oscillate, so it is strictly monotone, either strictly increasing or decresing.
case 1) if a<c<b but f(a)<f(b)<f(c)
then f(a)<f(b)<f(c) => a<b<c if it is strictly increasing which is a contradiction.
also f(a)<f(b)<f(c) => c<b<a if it is striclty decreasing which is also a contradiction.
case 2) if if a<c<b but f(c)<f(a)<f(b)
then f(c)<f(a)<f(b) => c<a<b if it is strictly increasing which is a contradiction.
also f(c)<f(a)<f(b) => b<a<c if it is striclty decreasing which is also a contradiction.
Hence if a<c<b, then f(a)<f(c)<f(b), and the function is strictly increasing.

Is it about right?
 
  • #8
Well I wouldn't have used the monotonicity because, while it's clear that it's true, we haven't proved it. In essence this theorem is equivalent to monotonicity, so it seems like a case of assuming what is to be proven.

My idea was to take a<c<b, and f 1-1. Since f is 1-1, f(c) cannot be equal to f(a) or f(b). First we assume that f(c)>f(b). Then we consider u such that f(b)<u<f(c), so IVT tells us that there is an x in [c,b] such that f(x) = u. However f(a)<u<f(c), so there is an y in [a,c] such that f(y) =u. But f was 1-1 and x and y are in different sets, so we have a contradiction. We conclude that f(c)<f(b). By repeating the argument for f(c)<f(a), we show that f(a)<f(c).
 
  • #9
Thanks so much for the clarification fzero!
 

What does "Prove that f(a) < f(c) < f(b)" mean?

This statement is a mathematical inequality that states that the value of the function f at the input a is less than the value of the function at the input c, which is also less than the value of the function at the input b.

Why is it important to prove this statement?

Proving this statement is important because it allows us to demonstrate the behavior of the function f at different input values. It also helps us understand the relationship between different inputs and outputs of the function.

What assumptions are needed to prove this statement?

To prove this statement, we need to assume that the function f is continuous on the interval between a and b and differentiable on the interval between a and c and the interval between c and b. We also need to assume that the input values a, c, and b are in the correct order, with a being less than c and c being less than b.

How can we prove this statement?

This statement can be proven using the intermediate value theorem, which states that if a function is continuous on an interval between two points, then it takes on every value between the function values at those two points. We can also use the mean value theorem to prove this statement by showing that the derivative of the function between a and c and between c and b is positive, indicating an increasing function.

What are the applications of this statement?

This statement has many applications in mathematics, physics, and engineering. It is used to analyze the behavior of functions and to prove the existence of solutions to equations. It is also used in optimization problems to find the maximum or minimum value of a function. In physics, it is used to analyze the motion of objects and to determine the velocity and acceleration of an object at different points in time.

Similar threads

  • Calculus and Beyond Homework Help
Replies
9
Views
556
  • Calculus and Beyond Homework Help
Replies
1
Views
284
  • Calculus and Beyond Homework Help
Replies
1
Views
505
  • Calculus and Beyond Homework Help
Replies
5
Views
1K
  • Calculus and Beyond Homework Help
Replies
6
Views
391
  • Calculus and Beyond Homework Help
Replies
26
Views
2K
  • Calculus and Beyond Homework Help
Replies
2
Views
744
  • Calculus and Beyond Homework Help
Replies
2
Views
325
  • Calculus and Beyond Homework Help
Replies
15
Views
1K
  • Calculus and Beyond Homework Help
Replies
2
Views
246
Back
Top